PANCE exam f part2

Pataasin ang iyong marka sa homework at exams ngayon gamit ang Quizwiz!

A 53-year-old previously healthy woman visits her primary care physician to review the results from the biopsy obtained from the solitary 8 mm polyp discovered in her colon by a routine colonoscopy. The report confirms that this is a hyperplastic polyp. Her family history is negative for colon cancer. Which one of the following is the most appropriate interval for follow-up colonoscopy in this patient? A1 year B10 years C3 years D5 years

10 years Small hyperplastic polyps, which are defined as less than 10 mm in size, are not neoplastic and are considered low risk. Colorectal Cancer Screening USPTF (updated 2021): screen all age 45-75, select patients up to age 85 ACG: start screening at age 45 AAFP: start screening at age 50 Test options: colonoscopy every 10 years, fecal immunochemical test yearly, multitarget fecal DNA every 3 years, CT colonography every 5 years One first-degree relative with colon cancer or advanced polyp: colonoscopy at age 40 or 10 years earlier than the age at which relative diagnosed Familial adenomatous polyposis: dependent on subtype and risk factors, screening in late childhood-adolescence; at least every 1-2 years

A 54-year-old man presents to the clinic to establish care. His medical history includes diabetes mellitus, hypertension, COPD, and tobacco use disorder. Medications include enalapril, metoprolol, metformin, sitagliptin, formoterol, and as-needed albuterol. The patient does not recall receiving any vaccines in the past. Vitals reveal HR 94 bpm, RR 18/min, BP 122/85 mm Hg, T 98.8°F, and SpO2 95% on room air. Physical exam reveals a barrel-chested man in no acute distress without labored breathing. Which of the following vaccines is most appropriate for this patient to prevent pneumococcal disease? A10-valent pneumococcal conjugate vaccine B13-valent pneumococcal conjugate vaccine C20-valent pneumococcal conjugate vaccine D23-valent pneumococcal polysaccharide vaccine E7-valent pneumococcal conjugate vaccine

20-valent pneumococcal conjugate vaccine Indicated in adults ≥ 65 years or < 65 years with risk factors -->For those who have not previously received PCV (or vaccine status unknown): One dose of PCV20 or PCV15 should be given -->If PCV 15 is given, administer PPSV23 at least 1 year later (except in patients with certain risk factors where an interval ≥ 8 weeks can be considered) -->For those who previously received PCV13 but not PPSV23: Administer PPSV23 at least 1 year after PCV13 Vaccines are inactivated

Change fluids to 5% dextrose in 0.45% NaCl and decrease insulin drip dose Diabetes-Related Ketoacidosis Patient will have diabetes History of infection, ischemia (cardiac, mesenteric), iatrogenic (e.g. steroids), insulin deficit (poor control), intoxication/illegal (cocaine abuse) (five I's) Abdominal pain, vomiting, and fatigue PE will show fruity-smelling breath, dehydration, and AMS Labs will show hyperglycemia, ketonemia, and an anion gap metabolic acidosis Management Treat precipitating cause Correct volume depletion with NS, add dextrose to fluids once glucose is < 200 mg/dL Replete K+ deficit (usually falsely elevated), do not start insulin if K+ < 3.3 mEq/L IV insulin drip until anion gap closes Corrected sodium: add 1.6 mEq/L for each 100 mg/dL in serum glucose HHS = hyperglycemic hyperosmolar syndrome ---------------------------------------------------------------------------- vs The patient's anion gap and ketonuria are improving, so increasing the insulin infusion (C) is not necessary.

A 12-year-old girl with diabetes presents to the ED complaining of multiple episodes of vomiting associated with persistent nausea and generalized abdominal pain. She also complains of fever, chills, and dysuria. Vital signs include a BP of 94/60 mm Hg, RR of 26 breaths per minute, HR of 106 beats per minute, T of 100.94°F (38.3°C), and oxygen saturation of 98% on room air. Her mucous membranes are dry, and her abdomen is tender to palpation. A point-of-care capillary glucose machine reads "high." She is given a 2 L bolus of normal saline. Urinalysis is positive for ketones, glucose, nitrite, and leukocyte esterase with 20-30 WBCs. You suspect this patient is in diabetic ketoacidosis secondary to a urinary tract infection. After the normal saline bolus, you start her on an insulin drip at 5 U/hr and administer antibiotics for the UTI. Below are the most recent lab values. What is the most appropriate next step in management? AAdminister ampule of NaHCO3 with a 0.9% NaCl bolus and increase insulin drip dose BChange fluids to 5% dextrose in 0.45% NaCl and decrease insulin drip dose CChange fluids to 5% dextrose in 0.45% NaCl and increase insulin drip dose DChange fluids to 5% dextrose in 0.45% NaCl and stop the insulin drip

Acyclovir PO dx:Genital Herpes Simplex Sx: painful genital rash, may be asymptomatic PE: grouped erythematous shallow cluster of vesicles and lymphadenopathy ---progress to shallow ulcers---------------->>> Labs: multinucleated giant cells on Tzanck smear (poor sensitivity) Dx: tissue PCR or viral culture Most commonly caused by herpes simplex virus (HSV) type 2, but HSV-1 infections are increasing in frequency Tx: acyclovirPregnancy: acyclovir or valacyclovir for 7-10 days after primary infection and from 36 weeks to delivery

A 19-year-old man presents to the ED with pain along his penile shaft for the past 7 days. He reports a low-grade fever and myalgias and was sexually active with a new partner 10 days ago. He reports no penile discharge or dysuria. There is no inguinal adenopathy palpated on exam, but he has tender penile lesions, as shown in the image above. Which of the following is most likely to be an effective treatment? AAcyclovir PO BCeftriaxone IM CDoxycycline PO DPenicillin G IM

Epiglottitis Sx: rapid onset of fever, muffled voice, dysphagia, respiratory distress PE: sore throat, stridor, leaning forward, drooling, red swollen epiglottis on direct visualization Imaging: enlarged epiglottis (thumbprint sign) on lateral neck X-ray Most commonly caused by H. influenzae (decrease since Hib vaccine), Strep and Staph species Treatment is airway management and IV antibiotics (third-generation cephalosporin AND antistaphylococcal) What is the recommended technique to establish a definitive airway in patients with severe epiglottitis? Answer: Awake fiberoptic intubation in the operating room with the patient sitting up. -------------------------------------------------------------------------------- vs A peritonsillar abscess (C) is the most common deep facial infection in adults. Patients present with fever, sore throat, trismus, and a peritonsillar mass that displaces the soft palate and uvula. A retropharyngeal abscess (D) is mostly a condition in infants and children, although it has been described in adults. Patients present with fever, neck pain, difficulty talking (termed cri du canard or a duck-like voice), swallowing, and breathing. The oropharyngeal exam sh

A 20-year-old man presents with a sore throat, dysphagia and drooling. He was seen 2 days ago for the same concern, but now the pain is worse. On examination, there is no pharyngeal erythema or tonsillar exudate. You obtain the soft tissue neck radiograph as seen above. Which of the following is the most likely diagnosis? ABacterial tracheitis BEpiglottitis CPeritonsillar abscess DRetropharyngeal abscess

A 55-year-old man on dialysis presents with the above ECG. What treatment should be immediately administered? Calcium gluconate Hyperkalemia Confirm no hemolysis in blood sample History of kidney failure, DKA, rhabdomyolysis, tumor lysis Lethargy, weakness, paralysis PE: bradycardia, hypotension, cardiac dysrhythmia ECG: peaked T waves, prolonged PR, wide QRS Treatment is calcium gluconate, insulin, albuterol, bicarbonate (less effective), dialysis, oral potassium binders (new, still undergoing further investigation) Review medication list for possible causes

A 55-year-old man on dialysis presents with the above ECG. What treatment should be immediately administered?

Membrane stabilization is a critical first step Calcium is recommended for the treatment of moderate to severe hyperkalemia (> 6.5 mEq/L) where ECG changes are present or the risk of dysrhythmia is present. Calcium's effect is rapid but transient. It provides membrane stabilization within one to three minutes of administration. Calcium antagonizes the effects of hyperkalemia at the cellular leve Hyperkalemia Confirm no hemolysis in blood sample History of kidney failure, DKA, rhabdomyolysis, tumor lysis Lethargy, weakness, paralysis PE: bradycardia, hypotension, cardiac dysrhythmia ECG: peaked T waves, prolonged PR, wide QRS Treatment is calcium gluconate, insulin, albuterol, bicarbonate (less effective), dialysis, oral potassium binders (new, still undergoing further investigation) Review medication list for possible causes ---------------------------------------------------------------------------- vs Calcium (B) does not lower the serum potassium level, but it stabilizes the cardiac membrane by restoring the normal gradient of the resting membrane potential of the cardiac cells

A 57-year-old man presents to the ED for shortness of breath. He has a history of hypertension, diabetes, and end-stage renal disease. His ECG is seen above. Which of the following is true regarding his management? AA normal ECG rules out hyperkalemia BCalcium chloride administration will decrease the serum potassium concentration CMembrane stabilization is a critical first step DRepeated doses of IV sodium bicarbonate are recommended until the QRS complex narrows

The shoulder is most vulnerable to an anterior glenohumeral dislocation in which of the following positions? AAbduction and external rotation BAbduction and internal rotation CAdduction and external rotation DAdduction and internal rotation A 24-year-old man presents with right shoulder pain after a fall onto his outstretched hand. A radiographic demonstrates an anterior inferior articular glenoid fracture. What is the name of this radiographic finding? ABankart lesion BHill-Sachs lesion CLabral lesion DLuxatio erecta

Abduction and external rotation Bankart lesion Anterior Shoulder Dislocation PE will show arm abducted, externally rotated, apprehension test positive X-ray will show humeral head displaced inferiorly and medially Diagnosis is made by anteroposterior (AP) and axillary shoulder X-ray Most commonly caused by trauma from a fall or forceful throwing motion Complications: axillary nerve damage, Bankart lesion, Hill-Sachs deformity

A 12-year-old boy with Wolff-Parkinson-White syndrome presents with palpitations for the past hour. His blood pressure is 110/62 mm Hg and pulse is 166 bpm. The ECG reveals a narrow-complex tachycardia. Vagal maneuvers are ineffective. Which of the following is the next most appropriate management? AAdenosine BCardioversion CProcainamide DRadiofrequency ablation

Adenosine Orthodromic (narrow complex): vagal maneuvers, AV nodal blockers (adenosine, beta-blocker, calcium channel blocker) Antidromic (wide complex): procainamide Synchronized cardioversion for hemodynamically unstable patients Definitive treatment is radiofrequency ablation

A 22-year-old man with a recent diagnosis of schizophrenia is brought by his parents for evaluation of abnormal behavior. His mother notes that he has been very restless and has been pacing around the house constantly. On examination, his vital signs are unremarkable. He is noted to be mildly agitated and rocking back and forth on the examining table. What is the likely diagnosis? AAkathisia BDystonic reaction CParkinsonian syndrome DTardive dyskinesia

Akathisia reversible extrapyramidal syndrome characterized by a subjective sensation of motor restlessness. It occurs most frequently with high-potency dopamine-2 receptor antagonists (e.g. haloperidol, fluphenazine, and thiothixene) Akathisia occurs within hours to days of starting the offending medication or when increasing the dose. Examples of possible behaviors include pacing the room, rocking back and forth, and repetitively crossing and uncrossing their legs. This behavior can be misconstrued as worsening agitation related to the underlying psychiatric illness. Diphenhydramine or benztropine (orally for mild symptoms, parenterally for more severe symptoms) are both effective in reducing acute symptoms.

A 55-year-old female is awaiting adrenalectomy due to a pheochromocytoma. Prior to surgery, which of the following classes of medications is most likely to be used to control hypertension? AAlpha-adrenergic blockers BAngiotensin converting enzyme inhibitors CAngiotensin receptor blockers DDiuretics

Alpha-adrenergic blockers Treatment involves using alpha blockers such as phenoxybenzamine. Beta blockers can be added following the administration of alpha blockers, but should not be used alone due to the possibility of worsening hypertension. Adrenalectomy is the definitive surgical treatment. Pheochromocytoma Catecholamine-secreting tumor located in the adrenal glands Sx: paroxysmal headaches, diaphoresis, palpitations, tremors, and vision changes PE: hypertension, orthostasis Dx: ↑ 24h urinary catecholamines and metanephrines, or ↑ plasma metanephrine levels, adrenal CT or MRI Tx :alpha-blocker (phentolamine, phenoxybenzamine) prior to beta-blockade to prevent unopposed alpha-agonism surgical resection Associated with MEN2 (medullary thyroid cancer, pheochromocytoma, +/- primary hyperparathyroidism) ↑ 24h urinary catecholamines be performed in a patient who is at low risk for a pheochromocytoma. vs high risk = plasma metanephrine levels

A 43-year-old man presents to the clinic for evaluation of perirectal pain that began last week. He states the pain has progressively worsened and has recently become so severe he cannot sit for more than a few minutes at a time. His medical history includes hypertension, diabetes, and hyperlipidemia, for which he takes metoprolol, enalapril, metformin, and simvastatin. Vital signs include HR 103 bpm, RR 17 breaths/min, BP 141/88 mm Hg, T 100.9°F, and SpO2 99% on room air. Physical examination reveals an erythematous patch of induration at the 2 o'clock perianal position. Primary intervention for this condition is undertaken with no complications. Which of the following is the best next step in the management of this patient's condition? AAmoxicillin-clavulanate BColonoscopy CComputed tomography of the abdomen and pelvis DMethylprednisolone ENo further management is required

Amoxicillin-clavulanate dx:anorectal abscess Prompt surgical drainage is required for all perianal and perirectal abscesses. Anorectal abscesses that are undrained can continue to propagate and may progress to generalized systemic infection. Surgical skin incisions should be made close to the anal verge to reduce the likelihood of fistula formation. Following drainage, clinicians often pack the wound with sterile gauze, but there has been no proven benefit to this practice. Patients who undergo incision and drainage of an anorectal abscess should be prescribed a 4- to 5-day course of amoxicillin-clavulanate or ciprofloxacin in combination with metronidazole if they have a history of immunosuppression, diabetes, valvular heart disease, signs of systemic infection, or extensive cellulitis.

A patient is scheduled for a unilateral adrenal tumor resection for suspected adrenocortical carcinoma. Which of the following surgical approaches would be best for this adrenalectomy? ALaparoscopic transabdominal BOpen transabdominal CPosterior retroperitoneoscopic DRetroperitoneal open

An open transabdominal approach is typically used for resection of any adrenal tumor when malignancy is suspected. The open approach allows for greater visualization of the surrounding structures and resection of the surrounding lymph nodes. The open approach also allows for resection of adjacent structures should this be necessary if abnormalities are discovered during the surgery. Minimally invasive techniques, such as laparoscopic techniques, have been associated with reduced blood loss and length of stay, fewer intraoperative complications, and quicker recovery than open approaches, however, when malignancy is known or suspected, the increased visualization and access to adjacent structures makes the open approach optimal. Minimally invasive techniques are the preferred method for benign adrenal tumors. ------------------------------------------------------------ vs Laparoscopic transabdominal (A) and posterior retroperitoneoscopic (C) approaches are both minimally invasive techniques and do not allow the visualization of or access to adjacent structures. The retroperitoneal approach (D) is not preferred because of the increased risk of subcostal neuropathy and lumbar hernias, as well as the increased amount of incisional pain from cutting through the latissimus dorsi muscle.

A 62-year-old man presents to the ED with crushing substernal chest pain for the last 2 hours associated with diaphoresis, nausea, and lightheadedness. His pain improves with sublingual nitroglycerin. Electrocardiogram obtained in triage shows deep Q waves and ST segment elevation in leads V1-V4, with ST depression in leads III and aVF. This pattern on ECG represents infarction of which area of the myocardium? AAnterior BInferior CLateral DPosterior

Anterior look at corresponding artery supply as well ----------------------------------------------------------- Posterior wall infarctions (D) are characterized by ST depressions in leads V1 and V2. Posterior leads can be placed on the patient's back, which will then reveal elevations

Which of the following statements is true regarding giant cell arteritis? AAortic involvement can lead to valvular disease and dissection BCorticosteroid therapy should be initiated only when biopsy confirms the disease CHistologic findings of inflammation are irreversible DIt is associated with sudden, painful binocular vision loss

Aortic involvement can lead to valvular disease and dissection --------------------------------------------------------------- vs Histologic findings (C) are rapidly reversed with steroid therapy. Temporal arteritis is a sight-threatening disease, but timely administration of high-dose corticosteroids can prevent blindness. For this reason, if the diagnosis is suspected, corticosteroids should be administered (B) immediately while awaiting the results of temporal artery biopsy. Symptoms suggestive of temporal arteritis include headache, jaw claudication, and visual disturbances. Temporal arteritis is associated with a sudden painless monocular loss of vision (D) due to vascular occlusion of the ophthalmic or posterior ciliary artery with infarction of the optic nerve or retina.

A 5-year-old girl presents to the emergency department with a 2-day history of grossly bloody diarrhea, abdominal cramping, and fever. Other children from her daycare have similar symptoms. On physical exam, the child appears mildly dehydrated and has mild lower abdominal tenderness to palpation. A stool study reveals fecal leukocytes and stool culture grows nonmotile, facultatively anaerobic, gram-negative rods. After initial fluid resuscitation, which of the following is the most appropriate treatment? AAzithromycin BCeftriaxone CCiprofloxacin DLoperamide

Azithromycin is the most appropriate treatment for shigellosis in patients < 18 years of age. Shigellosis is a type of bacterial diarrhea. In the United States, most cases of shigellosis are caused by Shigella sonnei. Treatment is based on resistance patterns but commonly azithromycin, ciprofloxacin, third-generation cephalosporin Consider Zinc or Vitamin A if malnourishment is present Complications: HUS, reactive arthritis For children with underlying immunodeficiency or who have toxemia or bacteremia, intravenous ceftriaxone is recommended. Azithromycin is the first-line oral treatment in children. Fluoroquinolone antibiotics are first-line treatment in adults. Complications of shigellosis include rectal prolapse, intestinal obstruction, colonic perforation, bacteremia, electrolyte imbalances, leukemoid reactions, and hemolytic-uremic syndrome. . The average incubation period is 3 days. Presenting symptoms may include abdominal pain, fever, mucoid or bloody diarrhea, and tenesmus. Stool frequency is usually 8-10 per day and significant fluid loss is not common. Diagnosis is based on small, volume bloody diarrhea, abdominal cramps, fever and with stool studies. Direct microscopy often reveals fecal leukocytes and red blood cells. Culture reveals non-motile, facultatively anaerobic, gram-negative rods that do not ferment lactose.

A 45-year-old woman presents to the clinic with urinary incontinence. She reports she has a sudden need to urinate but is unable to make it to the restroom before she voids. Vital signs include HR of 80 bpm, BP of 120/80 mm Hg, RR of 20/minute, oxygen saturation of 98% on room air, and T of 98.6°F. Physical examination reveals a regular rate and rhythm and a soft and nontender abdomen. Urinalysis reveals no leukocyte esterase or nitrites. Which of the following is the most appropriate next step in management? AAlpha-adrenergic agonist BAlpha-adrenergic antagonist CBladder training DContinence pessary ESerotonin and norepinephrine reuptake inhibitor

Bladder training dx:Urge Incontinence Most common cause of incontinence in older adults Sudden urgent need to urinate, not able to make it to the bathroom in time, nocturia Most commonly caused by overactive bladder (detrusor overactivity) Management includes: timed voiding, bladder training, anticholinergics, beta-3 agonists, pelvic floor exercise What lifestyle modifications are recommended for women with incontinence? Answer: Weight loss (for patients with obesity), cessation of cigarette smoking, and reduction of alcohol, caffeine, and carbonated beverage intake. Bladder training is an important nonpharmacologic treatment of urgency urinary incontinence. The patient should be instructed to void at specific times and attempt to gradually increase the time interval between each void. In addition to bladder training, there are pharmacologic therapies that can improve urgency urinary incontinence. These include antimuscarinics, such as oxybutynin, and beta-agonists, such as mirabegron. Perimenopausal and postmenopausal women with urgency incontinence and vaginal atrophy should be treated with topical estrogen. Women with urgency incontinence who do not improve with nonpharmacologic and pharmacologic interventions may be referred for surgery. ---------------------------------------------------------------- vs Alpha-adrenergic agonists (A), such as phenylpropanolamine, stimulate urethral smooth muscle contraction, which increases urethral resistance to urinary flow. Alpha-adrenergic agonists were previously used to treat stress incontinence but are no longer used because they are only mildly efficacious and associated with several adverse effects. Alpha-adrenergic antagonists (B), such as tamsulosin, are not used in the treatment of urgency incontinence. These medications are used to treat benign prostatic hyperplasia in men. Continence pessary (D) is a support device that can be used in the treatment of stress incontinence. It may be used as a substitute or ideally as an adjunct to pelvic floor muscle exercises. Continence pessaries do not have a role in the treatment of urgency incontinence. Serotonin and norepinephrine reuptake inhibitors (E), such as duloxetine, may be beneficial in the treatment o

A 27-year-old woman with no reported medical history presents to the emergency department after falling onto her left shoulder while playing soccer. Vital signs today include a heart rate of 70 bpm, blood pressure of 120/80 mm Hg, respiratory rate of 22 breaths per minute, pulse oxygenation of 99% on room air, and temperature of 98.6°F. Physical examination reveals a regular heart rate and rhythm. She has tenderness, edema, and bruising at the mid-aspect of her left clavicle. Her sensation is intact in her bilateral extremities, and her radial pulse is noted to be decreased on the left side. A plain radiograph shows a displaced fracture of the left clavicle. Which of the following is the best next step in management? AAdmission for open reduction and internal fixation BCompartment pressure measurement CCT angiogram of the left upper extremity DDischarge home with an arm sling and orthopedic follow-up EVenous ultrasound of the left upper extremity

CT angiogram of the left upper extremity dx:clavicle fracture and a decreased radial pulse on the left side, which is concerning for an associated vascular injury. Injury or compression of the left subclavian artery is a rare complication of clavicle fractures. The correct next step to confirm the suspected diagnosis and determine anatomic detail is a CT angiogram of the arteries of the left upper extremity. "specifically arterial thoracic outlet syndrome." Thoracic outlet syndrome refers to a group of disorders defined by compression of neurovascular structures as they pass through the thoracic outlet, which is between the clavicle and first rib at the base of the neck. Thoracic outlet syndrome is divided into three types (neurogenic, venous, and arterial) based on the structure being compressed (brachial plexus, subclavian vein, and subclavian artery). ----------------------------------------------------------------- vs Admission for open reduction and internal fixation (A) is not the most appropriate next step because the patient has signs of neurovascular compromise that need to be addressed immediately. Discharge home with an arm sling and orthopedic follow-up (D) is the correct management for closed clavicle fractures in patients without neurovascular compromise, respiratory distress, or hemodynamic instability. Venous ultrasound of the left upper extremity (E) is the correct diagnostic test for patients with clinical manifestations concerning for venous thoracic outlet syndrome. Venous thoracic outlet syndrome is often associated with repetitive overhead movements and causes extremity swelling, bluish discoloration, and often deep vein thrombosis.

A 33-year-old man presents with right eye pain, swelling, and fever for 2 days. Physical examination reveals upper and lower lid erythema and swelling. The patient has difficulty opening his eyelid and is unable to move the right eye laterally. What management should be initiated? ACT scan of the orbit and intravenous antibiotics BOral antibiotics and ophthalmology follow-up CTopical antibiotics and ophthalmology follow-up DWarm compresses A 3-year-old boy presents with atraumatic eye pain. Examination reveals a bulging, erythematous, edematous left eye with decreased extraocular movement and surrounding eyelid edema but normal intraocular pressure. Which of the following is the most appropriate initial management step? AConsult infectious disease service BConsult ophthalmology CIntravenous antibiotics DOral antibiotics

CT scan of the orbit and intravenous antibiotics dx: Orbital Cellulitis

Which of the following antiepileptic medications prevents seizures by blocking neuronal sodium channels to prolong the refractory period and inhibit action potential propagation? ACarbamazepine BDiazepam CEthosuximide DLevetiracetam

Carbamazepine is an effective, broad-spectrum antiepileptic that works primarily through sodium channel inhibition Common adverse effects of sodium channel blockade include drowsiness, ataxia, and diplopia. Like other anticonvulsants, carbamazepine can induce seizures in acute or chronic overdoses, so it is important to monitor serum drug concentrations and keep them between 6-12 mcg/mL. ----------------------------------------------------------------- vs Diazepam (B) is a benzodiazepine that enhances the binding of gamma-aminobutyric acid (GABA) to GABA-A channels to increase chloride influx. This increase causes neuronal hyperpolarization and decreased action potential propagation. Ethosuximide (C) works by blocking neuronal T-type calcium channels. Ethosuximide has a narrow spectrum of activity and is the drug of choice for absence seizures. Levetiracetam (D) works through a novel and not fully understood mechanism. Studies suggest that levetiracetam binds to the synaptic vesicle protein SV2A to modulate the release of neurotransmitters, such as glutamate and gamma-aminobutyric acid.

A 42-year-old woman presents with severe facial pain. The patient reports she has had episodes of severe, shooting pain on the lower half of the left side of her face. Her neurologic examination is normal. Her medical history is positive for herpes zoster. Which of the following is the appropriate next step? ACarbamazepine prescription BCT scan of the head CIbuprofen, acetaminophen, and prochlorperazine prescription DPrednisone prescription

Carbamazepine prescription Trigeminal Neuralgia (Tic Douloureux) Patient presents with sudden unilateral electric shock-like pains in gums, cheek, chin, temporal forehead PE: pain in V2 and V3 distributions, less commonly in V1 Tx: carbamazepine

A 6-year-old girl presents to the emergency department after being bitten on the dorsal aspect of her right hand by her neighbor's cat. The cat was found to be up-to-date on all of its vaccinations. After the wound was cleaned, the patient required suturing. The patient has an allergy to penicillin with rash and urticaria as the stated reaction. What prophylactic antibiotics are most suitable for this patient? AAmoxicillin and clavulanic acid BCefuroxime and clindamycin CCephalexin and metronidazole DSulfamethoxazole and trimethoprim

Cefuroxime and clindamycin Cat Bite Most commonly caused by Pasteurella multocida Treatment is irrigate, leave the wound open, amoxicillin-clavulanate Complications: osteomyelitis, tenosynovitis

An 84-year-old woman presents to the emergency department after she was struck from behind in a motor vehicle collision. She was wearing her seat belt and describes a sudden flexion and extension of her neck. She reports pain in her neck radiating to her arms, tingling in her hands, and difficulty walking. She states that prior to the injury she had some tingling in her hands and difficulty with fine motor tasks, which she attributed to arthritis, but now the symptoms are worse. On physical examination, she is alert and cooperative. She has diffuse weakness and loss of deep tendon reflexes in her upper extremities and intact motor function in her lower extremities. Sensation is reduced in the upper compared with lower extremities. Which of the following is the patient's most likely diagnosis? AAnterior cord syndrome BBrown-Séquard syndrome CCauda equina syndrome DCentral cord syndrome EPosterior cord syndrome

Central cord syndrome Central Cord Syndrome Most common incomplete spinal cord syndrome Hyperextension injury Bilateral motor and sensory deficits Upper extremity > lower extremity

A 24-year-old man presents to clinic today reporting redness and swelling to his left lower extremity. He has no significant medical history, takes no medications, and has no allergies. His symptoms started 2 days ago after hitting his leg at work. There is a small puncture wound associated with the swelling but no signs of purulent drainage. The wound appears edematous, erythematous, and warm to the touch with an orange peel appearance. He has not had a fever or signs of a systemic infection. Which of the following antibiotic regimens is the most appropriate? ACephalexin BCiprofloxacin CClindamycin DSulfamethoxazole-trimethoprim

Cephalexin dx: Cellulitis first-generation cephalosporin, such as cephalexin,provides adequate coverage of both Streptococcus and methicillin-susceptible Staphylococcus aureus and is the most appropriate treatment.

Which of the following is the most appropriate first step in caring for a patient who presents with an ear laceration? AChecking the patient for signs of altered consciousness and basilar skull fracture BIrrigation of the wound with normal saline CLocal infiltration of lidocaine with epinephrine DLocal infiltration of lidocaine without epinephrine

Checking the patient for signs of altered consciousness and basilar skull fracture ---------------------------------------------------------------- vs Once brain injury and basilar skull fracture are ruled out, the next most appropriate step would be (C) local infiltration of lidocaine with epinephrine. Previously, using epinephrine on areas such as the nose, ears, digits, and penis, was contraindicated due to concerns of necrosis, but that recommendation is outdated. (D) Infiltrating with lidocaine without epinephrine is still appropriate in patients with a lidocaine sensitivity, peripheral artery disease, or Raynaud phenomenon. Once anesthetized properly, the wound would be ready for (B) irrigation with normal saline, followed by closure with 5-0 or 6-0 nonabsorbable sutures.

A 45-year-old man with a history of poorly controlled diabetes mellitus presents with two weeks of otalgia despite treatment with topical antimicrobials. The pain is described as severe and he appears uncomfortable. The affected ear is erythematous, edematous, and appears more prominent than the unaffected ear. Which of the following is the most appropriate next step in management? AComputed tomography scan of the head with intravenous contrast BDischarge home with a prescription for oral antibiotics CLumbar puncture and cerebrospinal fluid analysis DTympanocentesis

Computed tomography scan of the head with intravenous contrast Necrotizing (Malignant) Otitis Externa

Which of the following is consistent with a diagnosis of bulimia nervosa? AContraction alkalosis BHeart failure CHyperkalemia DHypothermia

Contraction alkalosis

A 15-year-old boy presents with acute onset, atraumatic unilateral eyelid swelling. Examination reveals supratemporal orbital edema and tenderness. Which of the following is the most likely diagnosis? ADacryoadenitis BDacryocystitis CGraves disease DHorner syndrome

Dacryoadenitis Lacrimal gland enlargement Unilateral Severe pain, swelling Superolateral orbital region ---------------------------------------------------------------- vs Dacryocystitis (B) is an infectious obstruction of the nasolacrimal duct. Onset is usually rapid, with a clinical presentation of unilateral, severe pain, redness, and epiphora (overflow of tears). This condition occurs in the inframedial region of the orbit

A 64-year-old man presents to the clinic for management of chronic symptomatic hypercalcemia of malignancy. Despite the continued use of bisphosphonates, his total calcium level remains elevated. Which of the following is the most appropriate medication this patient should receive next? ACalcitonin BCinacalcet CDenosumab DZoledronic acid

Denosumab dx:Hypercalcemia Sx: bone pain (bones), kidney stones (stones), abdominal pain (groans), lethargy, psychosis (psychiatric overtones) ECG: shortened QT interval Most common causesMalignancy (most common inpatient cause) Primary hyperparathyroidism (most common outpatient cause) Treatment: IV fluids, bisphosphonates, calcitonin, denosumab (refractory disease) ---------------------------------------------------------------- vs Calcitonin (A) is used in the acute treatment of hypercalcemia. It works to reduce the serum calcium concentration by increasing renal calcium excretion and decreasing bone resorption by interfering with osteoclast function. The use of calcitonin is only beneficial during the first 48 hours of severe hypercalcemia and thus plays no role in long-term management.

A 65-year-old woman presents to your office with intertrigo in the axilla. The patient has obesity and diabetes mellitus type 2 and takes metformin. On examination, you detect small, reddish-brown macules that are coalescing into larger patches with sharp borders. A Wood lamp examination reveals coral-red fluorescence. You suspect a secondary infection complicating the intertrigo. What is the most appropriate topical treatment for this infection? AErythromycin BMupirocin CNystatin DSelenium sulfide EZinc oxide

Erythromycin dx:Erythrasma * Corynebacterium minutissimum is caused by Corynebacterium minutissimum and presents as small reddish-brown macules that may coalesce into larger patches with sharp borders. They may be asymptomatic or pruritic and fluoresce coral-red on Wood lamp examination. Intertrigo complicated by erythrasma is treated with topical or oral erythromycin. ---------------------------------------------------------------- vs Selenium sulfide (D) is an antifungal used as a first-line treatment for tinea versicolor. Tinea versicolor is commonly found on the trunk and can present as changes in pigmentation and a fine scale. The intertriginous areas are infrequently involved. Under a Wood lamp, a yellow-green fluorescence will be noted.

You are evaluating an intellectually challenged 45-year-old man for groin pain. On examination you notice that he has large ears, a prominent jaw, and large symmetric testicles. These findings are consistent with which one of the following? AAsperger's syndrome BFragile X syndrome CKlinefelter's syndrome DVariant form of Down syndrome

Fragile X X-linked Developmental delay Intellectual disability Dysmorphic features Long face, prominent jaw, large testicles, connective tissue disorders FMR1 gene mutation Supportive care Characteristic features during adolescence are an elongated face, prominent jaw, large ears, macro-orchidism, and a range of behavioral anomalies and cognitive deficits. Behavior is characterized by attention deficits, hand flapping, hand biting, and gaze aversion. The diagnosis of fragile X syndrome is confirmed by molecular genetic testing of the FMR1 gene. ----------------------------------------------------------- vs Asperger's syndrome (A) is a variant of autism in people of normal to high intelligence. Physical findings are insignificant however delays in social interactions, and restricted and repetitive behaviors are characteristics of this disorder. Down syndrome (D) is a chromosomal abnormality known as trisomy 21 and has a collection of clinical features which include micrognathia, macroglossia, epicanthal folds, flattened facial appearance, single transverse palmar crease as well as many other distinguishing features. This patient does not have any features of Down syndrome. Klinefelter syndrome (C) is caused by an additional X chromosome in males (47, XXY). Clinical findings are nonspecific during childhood; thus, the diagnosis commonly is made during adolescence or adulthood in males who have small testes with hypergonadotropic hypogonadism and gynecomastia. Virtually all men with Klinefelter syndrome are infertile.

A 58-year-old man ran out of his congestive heart failure medications. He presents with significant dyspnea and altered mental status. Examination reveals bibasilar crackles and jugular venous distension. An electrocardiogram shows sinus rhythm with low voltage complexes. Which of the following is most appropriate at this time? AAmiodarone BDisopyramide CFurosemide DVentricular assist device

Furosemide dxAcute or decompensated congestive heart failure BPAP: ↑ oxygenation, ↓ work of breathing, ↓ preload, afterload Nitroglycerin: ↓ preload, afterload Furosemide: diuresis Hypotension without signs of shock: dobutamine (may worsen hypotension) Severe hypotension with signs of shock: norepinephrine (↑ systemic vascular resistance, ↑ HR, ↑ BP, ↑ myocardial O2 demand)

You examine a two-day-old boy in the neonatal intensive care unit. He was born to a 26-year-old G2, P2 mother via normal spontaneous vaginal delivery. The mother had limited prenatal care. The nurse notes that the infant has a poor suck. On examination, you note that the neonate is small for gestational age, with microcephaly, enlarged liver, and petechiae. On complete blood count, you note thrombocytopenia. You suspect congenital cytomegalovirus infection. Which of the following is the most appropriate therapy? AAcyclovir BCidofovir CFoscarnet DGanciclovir

Ganciclovir Congenital Cytomegalovirus Rash (petechiae, purpura) Microcephaly IUGR Hepatosplenomegaly Seizures Retinitis or vision loss Hearing loss, baby will need follow-up hearing tests Dx: urine, saliva, blood, CSF for CMV DNA (PCR) Tx: oral valganciclovir or IV ganciclovir (symptomatic disease)

A 57-year-old type 2 diabetic man presents to his primary care provider for his annual physical. He was diagnosed with diabetes seven years ago and has a history of alcohol use disorder. His most recent hemoglobin A1C was 7.9 percent and fasting glucose of 156. He is not on any diabetic medications. He states that two years ago he had an episode of intense mid-abdominal pain that radiated to his back. He has had two more similar episodes. Ever since then, he has constant epigastric pain and has noticed loose, foul smelling stools that float in the toilet. Considering his glycemic control and past medical history, what class of anti-hyperglycemic agents is contraindicated for this patient? ABiguanides BGlucagon-like peptide-1 agonists CSulfonylureas DThiazolidinediones

Glucagon-like peptide-1 agonists dx: history of pancreatitis and should be used with caution in patients with pancreatitis risk factors (alcoholism, biliary disease). ---------------------------------------------------------- vs Biguanides (A), such as metformin, are contraindicated if creatinine is greater than 1.5, in lactic acidosis, and in hepatic impairment. Biguanides should be discontinued 24 hours prior to iodinated contrast administration and resumed 48 hours afterwards with monitoring of creatinine. Metformin is usually the first-line oral medication used to control type II diabetes and would likely be the best choice for this man. Sulfonylureas (C), such as glipizide and glyburide, have the potential to cause hypoglycemia and should not be used in patients with a sulfonamide allergy or G6PD deficiency. Thiazolidinediones (D) are contraindicated in patients with New York Heart Association class III and IV cardiac status due to potential adverse effect of edema and worsening of congestive heart failure. Patients on thiazolidinediones should be monitored for excessive, rapid weight gain or dyspnea.

A 45-year-old man presents to his primary care clinician due to concerns that his chronic hemorrhoids are worsening. He describes increasing anorectal bleeding and pain. He feels a fullness in the area that is new over the past few months. He describes a sensation that there is something in the way of his bowel movements and during anal intercourse. His medical history is notable for HIV, hemorrhoids, and anorectal warts. His most recent CD4 count was 150 cells/mcL, which is the lowest his counts have been. His only medication is a combination antiviral pill which includes bicetegravir, tenofovir alafenamide, and emtricitabine. He is sexually active with multiple men. He reports that since his HIV diagnosis 10 years ago, he now uses condoms during intercourse. He reports that he did not use protection prior to his HIV diagnosis. Which of the following represents the most likely pathogenesis responsible for these findings? ACytomegalovirus BHerpes simplex virus type 2 CHerpes zoster DHerpesvirus 4 EHuman papillomavirus

Human papillomavirus

A 50-year-old G2P2A0 woman presents to the OB/GYN office for follow-up regarding her cervical cytology result. Her history is significant for hypertension, and she takes lisinopril 10 mg daily. She has been coming in for her routine Papanicolaou smear every 3 years with no abnormal results in the past. On her most recent cervical cytology, atypical squamous cells of unknown significance were found. Vital signs are a BP of 127/78 mm Hg, HR of 86 bpm, RR of 16/min, SpO2 of 98% on room air, and a T of 98.7°F. On pelvic exam, normally developed genitalia are noted with no external lesions or eruptions. The vagina and cervix show no lesions, inflammation, discharge, or tenderness. What is the best next step in evaluation of the patient's cytology results? AColposcopy BEndocervical curettage CHuman papillomavirus testing DRepeat cervical cytology in 2 years ERepeat cervical cytology today

Human papillomavirus testing , atypical squamous cells of unknown significance LOOK At chart in book

A 63-year-old woman presents with a facial pain for 1 week followed by a rash starting 2 days ago. She has a history of breast cancer and takes tamoxifen daily. Vital signs include a BP of 136/88 mm Hg, HR of 95 bpm, T of 98.5°F, and RR of 16/minute. Which of the following signs would indicate the suspected diagnosis has affected the ophthalmic branch of the trigeminal nerve? AAuspitz sign BCrowe sign CHutchinson sign DMarcus Gunn pupil ENikolsky sign

Hutchinson sign Herpes Zoster (Shingles) Risk factors: immunosuppressed, advancing age Caused by reactivation of latent varicella-zoster virus Sx: painful, papulovesicular rash preceded by tingling or hyperesthesia PE: rash with unilateral, dermatomal distribution that does not cross midline Dx: based on clinical presentation, PCR if confirmation needed (Tzanck smear shows multinucleated giant cells - not typically done) Tx: antiviral therapy (best if started within 72 hours) Postherpetic neuralgia: when pain persists beyond 3 months in the same distribution as a preceding documented episode of acute herpes zoster Prevention: RZV two-dose series ------------------------------------------------------------ vs Marcus Gunn pupil (D), or afferent pupillary defect, affects the optic nerve (cranial nerve II) and is most commonly caused by optic neuritis. It occurs when the pupil dilates instead of constricts in the affected eye when light is shown into it during the swinging flashlight test

A 35-year old man believes he has colon cancer. He reports that his "peristalsis is louder than usual" and he has "excessive flatulence." He shows you a logbook that he has been keeping of his bowel habits for the last 6 months. He reports no weight loss, fatigue, night sweats, blood in his stool, or family history of colon cancer. He is having difficulty sleeping. What is the most likely diagnosis? ABody dysmorphic disorder BFunctional neurological symptom disorder CIllness anxiety disorder DSomatic symptom disorder

Illness anxiety disorder Hypochondriasis Preoccupied with serious illness, despite negative exam or testing Evaluate for other medical diagnosis Management is CBT, psychotherapy, antidepressants ---------------------------------------------------------------- vs Somatic symptom disorder (D) is characterized by excessive worry and behaviors related to one or more (often multiple) somatic symptoms. Unlike illness anxiety disorder, patients are not focused on a particular illness. Symptoms may vary over time, but the disorder is persistent.

A 60-year-old man presents to the clinic for medication refills. He takes insulin glargine 20 units nightly, insulin lispro with each meal according to a sliding scale, lisinopril 10 mg daily, metformin 1,000 mg twice daily, and atorvastatin 10 mg daily. He brings in his blood glucose log, which shows evening postprandial blood glucose readings averaging 240 mg/dL and morning fasting readings averaging 350 mg/dL. The patient's vital signs today are within normal limits, except a blood glucose level of 300 mg/dL and a body mass index of 30 kg/m2. Hemoglobin A1C measurement today is 10.6%, which is 1% better than 3 months ago. Which of the following is the best clinical intervention for this patient? AAdd 8 units of insulin lispro to the sliding scale with each meal BAdd glimepiride 4 mg with evening meal CDecrease insulin glargine from 20 units at bedtime to 15 units at bedtime DIncrease insulin glargine from 20 units to 28 units at bedtime EIncrease metformin to 1,000 mg three times daily

Increase insulin glargine from 20 units to 28 units at bedtime The patient in the above vignette has type 2 diabetes mellitus. Patients with type 2 diabetes mellitus demonstrate insulin resistance and slowly diminishing pancreatic beta cell function This is called the dawn effect and is the result of physiologic increases in growth hormone release in the early morning hours that stimulate gluconeogenesis in the liver. In patients who do not have diabetes mellitus, more insulin is produced during the early morning growth hormone release. Patients with diabetes who are insulin-dependent need increases in their early morning insulin to counteract this phenomenon. As most of this hyperglycemia happens while the patient is asleep,

A 16-year-old boy presents to the pediatrician's office with his parent reporting left-sided scrotal discomfort and heaviness over the past 6 months. He describes a dull ache that worsens with coughing, which he noticed a few weeks ago after having a mild cold. His symptoms are exacerbated with extended standing and improved by lying supine. He has no chronic medical conditions and no surgical history. He reports no prior sexual intercourse but reports having a girlfriend over the past few months. He is considering becoming sexually active, which led him to be evaluated for his scrotal symptoms. He reports no nausea, vomiting, changes in urination, changes in bowel habits, or impotence. His vital signs are within normal limits, including being afebrile. Inspection reveals a low-hanging scrotum on the left side. Elevation of the scrotum relieves his pain, and the cremasteric reflex is intact and symmetric bilaterally. Valsalva testing increases his symptoms, and there is a thrill present when coughing. The scrotum does not transilluminate. You discuss with the patient and his parent about surgical correction for this condition. Which of the following is the most concerning complication if

Infertility Varicocele More common in adolescent males PE: scrotal mass (bag of worms) that disappears when recumbent Most commonly occurs on the left side - caused by an increase in venous pressure, causing dilation of pampiniform venous plexus along the spermatic cord Dx: only exam needed if left-sided and disappears when recumbent; if right-sided or present when supine, U/S needed to look for secondary causes Tx: monitoring, surgery if fertility affected and desired

A 24-year-old patient presents to the emergency department and reports severe right-sided lower abdominal pain that began suddenly about 30 minutes ago and is associated with nausea. She has a temperature of 98.4°F, blood pressure of 152/72 mm Hg, and heart rate of 122 bpm. A serum beta-human chorionic gonadotropin is negative. On physical exam, the patient has moderate abdominal tenderness from palpation to the lower right quadrant, with active bowel sounds. A Doppler pelvic ultrasound reveals a 6 cm ovarian mass with impaired blood flow to the ovary. Which of the following structures does the patient's condition involve pathology in? ABroad ligament BCardinal ligament CInfundibulopelvic ligament DRound ligament EUterosacral ligament

Infundibulopelvic ligament The ovary is suspended by the infundibulopelvic ligament, which is not fixed in place, and the presence of an ovarian mass predisposes the ovary to rotate on the axis of this ligament and the utero-ovarian ligament (which is more proximally located). The right ovary is more commonly affected than the left because the right utero-ovarian ligament is longer than the left. dx:Adnexal Torsion Twisting of ovary, fallopian tube, or a paraovarian mass Patient likely to be of reproductive age, mean age 29-33.5 years Sudden onset of unilateral (right > left) abdominal and pelvic pain Laboratory abnormalities are often absent, leukocytosis may be present with necrosis Imaging: pelvic U/SOvarian torsion Enlarged heterogeneous ovarian mass Peripheralization of the follicles may be present Definitive diagnosis and management: laparoscopic detorsion Risk factors include pregnancy, being of reproductive age, ovulation induction for infertility treatment (which may result in the formation of large ovarian cysts), and history of prior torsion. An ovarian mass ≥ 5 cm increases the risk of torsion. Examples of ovarian masses are ovarian cysts (e.g., functional cyst or corpus luteum in early pregnancy) and ovarian neoplasms ----------------------------------------------------------------- vs The broad ligament (A) helps hold the uterus in position and also carries blood vessels, nerves, and lymphatics to the structures within the mesentery that supply the uterus, fallopian tubes, and ovaries. The cardinal ligaments (B) are fibrous bands that attach the cervix to the lateral pelvic walls. The round ligaments (D) connect the anterior uterus and travel into the pelvis through the inguinal canal to attach to the labia majora. They help keep the uterus in an anteverted position. The uterosacral ligaments (E) attach the posterior cervix to the anterior surface of the sacrum.

An 8-year-old girl presents to your office with complaints of increased thirst, increased volume of urine and weight loss. Physical exam is normal except for the patient appearing slightly ill and with a weight loss of seven pounds since her last well child exam. Hemoglobin A1C is 9.2%. Which of the following is the most appropriate therapy? AGlipizide BInsulin CMetformin DRepaglinide

Insulin

A patient is diagnosed with multiple sclerosis. Imaging shows that the majority of plaques are located in the cerebellum. Examination is positive for dysmetria and ataxia. Which of the following types of tremor would most commonly occur in this patient? AIntention BPill-rolling CPostural DRubral

Intention

A 44-year-old previously healthy woman presents to the emergency room with unilateral facial pain that began several hours ago. She now has swelling in the left side of her face and tenderness in this area. Physical exam reveals left-sided parotid gland swelling, trismus, and purulent ductal discharge. Which of the following is the best pharmacologic treatment? AIntravenous linezolid BIntravenous nafcillin COral ciprofloxacin DOral doxycycline

Intravenous nafcillin dx: Sialadenitis Patient presents with mouth swelling and pain that is worse with meals Most commonly caused by Staphylococcus aureus Treatment is antibiotics, warm compresses, gland massage, sialogogs What is Ludwig angina? Answer: A severe submental and submandibular infection involving the floor of the mouth. ------------------------------------------------------------ vs Linezolid (A) is reserve for immunocompromised patients and can be used instead of vancomycin in some treatment regimens. Oral ciprofloxacin (C) and doxycycline (D) are not sufficient for this patient in the acute setting, but ciprofloxacin may be used as part of a step-down regimen once improvement with intravenous antibiotics has taken place. Some experts recommend oral antistaphylococcal agents for patients with sialadenitis who present with mild disease.

A 43-year-old man presents with altered mental status. His vital signs are HR 113 bpm, BP 143/63 mm Hg, T 98.9°F, and blood glucose of 750 mg/dL. During your evaluation, he has a brief generalized tonic-clonic seizure. Which of the following represents the most important initial intervention for this patient? AAlteplase BInsulin bolus and continuous infusion CIntravenous normal saline DPhenytoin

Intravenous normal saline Hyperosmolar Hyperglycemic State Patient most commonly has history of type 2 DM (has been reported in children with type 1 diabetes) AMS PE will show profound dehydration Labs will show glucose > 600 mg/dL and negative ketones Management includes IV fluids, may also need insulin or electrolyte replacement

A 19-year-old woman presents with pain in her mouth. She underwent an extraction of an impacted molar 3 days prior to the onset of her pain. Her pain had been improving, however, the pain began worsening acutely today. Which of the following is the most appropriate treatment? ADilute hydrogen peroxide rinses BIncision and drainage CIodoform gauze packing moistened with oil of clove DRoot canal

Iodoform gauze packing moistened with oil of clove Alveolar Osteitis (Dry Socket) Patient will be 3-5 days postextraction PE will show inflammation and local osteitis with a clean extraction site Most commonly caused by loss of healing clot Treatment is pack socket with iodoform gauze and eugenol oil What is expected on physical examination of dry socket? Answer: Exposed bone and no clot in the extraction site.

A 19-year-old woman at 34 weeks gestation presents with a 1-day history of progressive left-sided facial weakness. She notes a recent history of an upper respiratory infection. On exam, she is unable to raise her left eyebrow, close her left eye, or smile on her left side. She also reports decreased taste sensation. Her neurologic exam is otherwise unremarkable. Which of the following complications is most commonly associated with the patient's diagnosis? ACavernous sinus thrombosis BDescending paralysis CHearing loss DKeratitis

Keratitis Bell Palsy History of prodrome Waking up with unilateral facial nerve paralysis, hyperacusis, and taste disturbance PE: CN VII palsy that does NOT spare the forehead Commonly thought to be associated with HSV activation Tx: prednisone, artificial tears, tape eyelid shut, antivirals (for severe cases) Bilateral: Lyme disease, infectious mononucleosis

A 28-year-old man presents with scrotal swelling and pain. Examination reveals a left, nontender, swollen spermatic cord that feels like a "bag of worms." The swelling reduces when the patient moves to a supine position. There are no skin lesions, hernia or testicular abnormalities. In addition to scrotal ultrasonography, you may consider ordering further imaging with attention to which of the following organs? AAscending colon BDescending colon CKidney DUrinary bladder

Kidney Primary varicocele is usually idiopathic in nature. Secondary varicocele can occur due to abdominal mass compression of the renal veins (more common with right sided varicocele), or superior mesenteric artery compression of the left renal vein (aka "Nutcracker Syndrome"; most commonly due to renal cell carcinoma and retroperitoneal fibrosis/adhesions). Varicocele More common in adolescent males PE: scrotal mass (bag of worms) that disappears when recumbent Most commonly occurs on the left side - caused by an increase in venous pressure, causing dilation of pampiniform venous plexus along the spermatic cord Dx: only exam needed if left-sided and disappears when recumbent; if right-sided or present when supine, U/S needed to look for secondary causes Tx: monitoring, surgery if fertility affected and desired

A 9-year-old girl is in the clinic for a follow-up. She was diagnosed with asthma a few days ago after an ED visit. She has had night awakenings about twice a week. She has been using her inhaled short-acting beta agonist almost every day for relief of shortness of breath. On physical examination, you note occasional wheezing on both lung fields. Which of the following is the correct classification of this patient's asthma severity? AIntermittent BMild persistent CModerate persistent DSevere

Moderate persistent dx: asthma

A 60-year-old man with a history of hypertension and type 2 diabetes mellitus presents to your office for his annual exam. He reports nonadherence with his routine medications and has been taking ibuprofen daily for the past 6 months due to headaches. Which of the following is the most useful initial imaging study to evaluate this patient for chronic kidney disease? ACT scan with contrast BCT scan without contrast CKidney ultrasound DMRI

Kidney ultrasound Complications of CKD: Volume overload Electrolyte imbalance (hyperkalemia, hyperphosphatemia, hypocalcemia) Normocytic anemia: ↓ EPO + anemia of chronic disease Secondary hyperparathyroidism, mineral bone disorder Acidosis Platelet dysfunction, bleeding Cardiovascular disease and dyslipidemia Increased risk of infection ------------------------------------------------------------ vs CT scan with contrast (A) should be avoided in patients with suspected chronic kidney disease because radiographic contrast dye can induce kidney failure. CT scan without contrast (B) and MRI (D) are used to diagnose renovascular disease.

A 52-year-old man with a long history of alcohol use disorder presents with epigastric pain and vomiting. You diagnose him with pancreatitis. Which of the following laboratory values is associated with an increased risk of mortality on admission? AALT 350 mmol/L BLDH 400 IU/L CLipase 14,000 U/L DWBC 15,000 cells/mm3

LDH 400 IU/L Ranson Criteria At Admission Glucose > 200 mg/dL LDH > 350 IU/L AST > 250 SF units Age > 55 years WBCs > 16.000/mm3 Acute Pancreatitis Sx: epigastric pain radiating to the back, worse when lying down and better when leaning forward, nausea, and vomiting PE: flank ecchymosis (Grey Turner sign), umbilical ecchymosis (Cullen sign) Labs: elevated lipase (best) and amylase Ranson criteria and APACHE II are used to predict the severity (Note: they are difficult to apply and have limitations) Caused by gallstones > alcohol, hypertriglyceridemia, drugs Treatment is IV fluids, analgesics Complications: necrotizing pancreatitis, pancreatic pseudocyst, pseudoaneurysm

A 4-year-old boy presents to your office with watery nonbloody diarrhea for the last three months. He has occasional abdominal pain prior to the onset of the diarrhea and his mother noted worsening flatulence, especially after meal times. Mom denies any recent travel or camping and the patient does not go to school or daycare. On exam, he is 60th percentile for weight and height (similar to his last well-child check), appears well, and has a normal abdominal exam. What is the most likely diagnosis? ACeliac disease BInflammatory bowel disease CIrritable bowel syndrome DLactase deficiency

Lactase deficiency Diagnosis is usually clinical, lactose tolerance test, lactose breath hydrogen test Most commonly caused by a genetic condition or secondary to underlying conditions Treatment is lactose restriction, calcium, vitamin D What is the most common disaccharide enzyme deficiency? Answer: Lactase deficiency. ------------------------------------------------------------- vs Inflammatory bowel disease (B) (IBD) is characterized by bloody stools and failure to thrive. Irritable bowel syndrome (C) (IBS) is characterized by diarrhea that alternates with constipation. The associated abdominal pain is often relieved by defecation and there are no systemic features. Celiac disease (A) is associated with diarrhea, abdominal pain and distension, but is usually associated with a failure to thrive. Moreover, the condition is caused by a permanent gluten intolerance.

A 62-year-old woman presents with pain and swelling of the joints in her hands, wrists, and forefoot. She describes a stiff feeling in her hands that is worse in the morning and reports that her grip strength is decreased. Laboratory tests are positive for rheumatoid factor and anti-citrullinated peptide antibodies. Erythrocyte sedimentation rate and C-reactive protein tests are also elevated. After considering her diagnosis, the clinician decides to begin therapy with methotrexate. Which of the following is the most appropriate test to order prior to beginning this therapy? AAutomated visual field testing BChest radiograph CDilated retinal examination DInterferon-gamma release assay ELiver aminotransferase levels

Liver aminotransferase levels

A 30-year-old woman presents with shortness of breath. She has a history of asthma since childhood. She says that she uses her rescue inhaler 3 days each week. On average, she wakes up in the middle of the night with symptoms approximately 3 times a month. Her vital signs are BP 120/76 mm Hg, HR 112 bpm, RR 26/min, and oxygen saturation 92% on room air. The patient appears to be in mild distress with intercostal retractions present. The respiratory exam is remarkable for diffuse wheezing. This patient is diagnosed with an acute asthma exacerbation, and treatment is initiated. Which of the following best classifies this patient's asthma? AIntermittent BMild persistent CModerate persistent DSevere persistent

Mild persistent dx: asthma

A 3-year-old boy presents with his mother after he tripped and sustained an injury to his mouth. The mother has his avulsed tooth soaking in a container of liquid. Which of the following is true concerning the management of dental avulsions? AA primary central incisor should be reimplanted BA primary tooth avulsion often leads to a cosmetic deformity of the secondary tooth CAvulsed teeth should be held by their root to avoid contaminating the crown DMilk is an acceptable storage medium for an avulsed tooth

Milk is an acceptable storage medium for an avulsed tooth Tooth Avulsion Reimplant tooth as soon as possible Primary teeth: do not reimplant Transport in Hank solution (best), milk Rinse tooth (do not brush) ------------------------------------------------------------------ vs Reimplanted primary teeth (A and B) ankylose or fuse to the bone so that, although the dentofacial complex grows downward and forward, the reimplantation site does not. There also may be interference with the eruption of the permanent tooth. Hence, primary teeth should not be reimplanted. Avulsed teeth should be handled only by the crown (C) so the remaining periodontal ligament fibers are not damaged.

How does the presentation of abdominal pain differ in patients of advanced age? AMore likely to need an emergent surgical procedure BMore likely to present with an elevated white blood cell count CMore likely to present with fever DMore likely to present with peritoneal signs

More likely to need an emergent surgical procedure

A 50-year-old man presents with episodic severe vertigo lasting hours, with associated symptoms of unilateral tinnitus, fluctuating low-frequency hearing loss, and aural fullness. Which of the following is the most likely diagnosis? ABenign paroxysmal positional vertigo BMeniere disease CPerilymph fistula DVestibular neuronitis

Ménière Disease Patient presents with episodic low-frequency hearing loss, tinnitus with aural (ear) fullness, and vertigo lasting 20 minutes up to 24 hours Diagnosis is made clinically Most commonly caused by too much inner ear endolymph and increased pressure within the inner ear Treatment is low-salt diet, diuretics (HCTZ + triamterene)

A 24-year-old G1P0 woman presents to the emergency department for atraumatic vaginal bleeding at 12 weeks of gestation. The woman's vital signs are within normal limits. On examination, she is found to have blood in the vaginal vault with an open cervical os. Obstetrics is consulted. Rh(D) immunoglobulin is given to the patient after laboratory results are available. Which of the following test results indicated the patient's need for Rh(D) immunoglobulin? ADecrease in hCG by 40% BLow hemoglobin and hematocrit on complete blood count CNegative Rh(D) on blood typing and antibody screen DPositive Kleihauer-Betke test

Negative Rh(D) on blood typing and antibody screen Type and screen should be used to determine the mother's blood type. In instances of severe maternal fetal hemorrhage, Kleihauer-Betke test can measure the occurrence and severity of hemorrhage.it does not indicate the mother's Rh status to determine if Rh(D) immunoglobulin is indicated. Rh Isoimmunization Rh-negative mothers exposed to Rh-positive blood → anti-Rh antibodies Subsequent pregnancies: jaundice, anemia, fetal hydrops, fetal death Prevention: anti-D globulin at 28 weeks (and within 72 hrs of delivery if infant is Rh+) ------------------------------------------------------------------ vs Decrease in hCG by 40% (A) is incorrect. While a decreasing hCG greater than 35% is predictive of miscarriage, this result alone would not indicate a need for Rh(D) immunoglobulin administration. Low hemoglobin and hematocrit on complete blood count (B) is incorrect. While it is important to evaluate the mother for possible anemia and transfusion if necessary, the complete blood count would not provide a result indicating that Rh(D) immunoglobulin is necessary. Positive Kleihauer-Betke test (D) is incorrect. While it is used to detect fetal-maternal hemorrhage, it does not indicate the mother's Rh status to determine if Rh(D) immunoglobulin is indicated.

A 67-year-old man with a history of hypertension presents to the emergency department complaining of the acute onset of severe tearing chest and back pain and a syncopal episode. Which of the following physical exam findings would support the most likely diagnosis? AAnkle-brachial index of 1.0 BHamman crunch CMydriasis DNew murmur of aortic regurgitation EPositive Murphy sign

New murmur of aortic regurgitation Aortic Dissection Risk factors: advancing age, male sex, HTN, Marfan syndrome Sx: acute onset of "ripping" or "tearing" chest pain or back pain PE: asymmetric pulses or SBP difference of > 20 mmHg CXR: widened mediastinum Dx: CT angiography or transesophageal echocardiogram (TEE) Treatment: reduce BP and HR (beta-blockers), pain control, emergency surgery (Type A dissection) IV Labetalol or esmolol Type A: involves ascending aorta Type B: involves only descending aorta ------------------------------------------------------------- vs Hamman crunch (B) is an adventitious sound heard on auscultation of the chest that results from the presence of mediastinal air. It is a classic finding in the setting of esophageal rupture.

An 96-year-old woman is admitted to the hospital with multiple chronic conditions, including heart failure, chronic kidney disease, and diabetes. Her overall health is delicate, and she has a history of frequent hospitalizations over the past year. She is currently experiencing a severe respiratory infection and is dependent on supplemental oxygen for breathing support. She begins to show signs of respiratory distress, increased fatigue, and declining kidney function. The medical team suggests starting an aggressive course of intravenous antibiotics and initiating non-invasive mechanical ventilation to support her breathing. You have concerns that these interventions may be more harmful than beneficial for the patient given her advanced age, frailty, and the overall burden of her medical conditions. Which of the following ethical principles applies to your decision-making in this scenario? AAutonomy BBeneficence CDistributive justice DFidelity ENonmaleficence

Nonmaleficence Ethical Principles Autonomy: respect patient's right to decide on treatment options Beneficence: act in patient's best interest Nonmaleficence: refrain from actions that cause harm Justice: resource allocation to maximize benefits and minimize burdens for all patients, regardless of race, socioeconomic class, etc.

An 8-year-old girl presents with complaints of difficulty walking over the past few days. She initially had numbness and tingling in her hands and feet which then progressed to a heavy, weak feeling in her legs. Physical exam is notable for the absence of deep tendon reflexes. Which laboratory finding is most consistent with this patient's diagnosis? AElevated CSF WBC count, normal protein BElevated serum WBC count with significant lymphocytosis CHematuria and proteinuria DNormal CSF WBC count, elevated protein

Normal CSF WBC count, elevated protein Guillain-Barré Syndrome Risk Factors: recent minor respiratory or GI illness Sx: Symmetric, progressive ascending muscle weakness, can lead to respiratory failure, often associated with dysautonomia PE: lack of deep tendon reflexes, symmetric weakness Lumbar puncture: increased CSF protein but a normal cell count Most commonly caused by Campylobacter jejuni Treatment is supportive, IVIG, or plasmapheresis

A 16-year-old girl presents to the ED for a minor laceration repair of her forehead after a picture frame fell off the wall and hit her. Her vital signs are blood pressure 175/75 mm Hg, HR 80 bpm, and RR 14/min. The patient states on review of systems that she has had headaches, chest pain, and fatigue over the previous few months. You note a systolic murmur in the left infraclavicular area and under the left scapula. Which of the following is the most important next step in management? AArrange for outpatient follow-up for repeat blood pressure BObtain blood pressure readings in the upper and lower extremities CObtain urinary metanephrines DOrder a renal ultrasound with Doppler

Obtain blood pressure readings in the upper and lower extremities dx: Coarctation of the Aorta Narrowing of descending aorta Sx: headaches, dyspnea, leg claudication PE: higher blood pressure in the arms than in the legs, delayed femoral pulse ECG: LVH CXR: notching of ribs, "3 sign" (indentation of aorta) Dx: echo Tx: balloon angioplasty with stent placement or surgical correction Associated with Turner syndrome, bicuspid aortic valve, and intracranial aneurysms

A 36-year-old woman presents with numbness of her arms and legs for 2 hours. She is status post total thyroidectomy 3 days prior to presentation and is on thyroid hormone replacement therapy. Vital signs are normal and the patient's exam is significant for a clean, well-healing surgical incision on her neck and twitching at the corners of the mouth with tapping at the angle of the jaw. After intravenous repletion of the suspected electrolyte disorder the patient remains persistently low. What is the next step in management? AObtain a non-contrast CT scan of the head BObtain magnesium level CReplete potassium DStop thyroid hormone replacement

Obtain magnesium level dx:If a patient remains persistently low despite adequate intravenous calcium repletion, magnesium levels should be obtained and supplemented as needed. Hypocalcemia Sx: seizures, paresthesias PE: Chvostek sign (facial muscle contraction), Trousseau sign (carpopedal spasm), hyperreflexia Labs: serum Ca2+ < 8.5 mg/dL ECG:↑ QT interval Tx: IV replacement for calcium ≤ 7.5 mg/dL (1.9 mmol/L) or tetany, seizure, carpopedal spasm, prolonged QT (mag calcified low)nemonic

A 5-year-old boy is brought to the ED by his mother for leg pain. He was playing with his sister when he fell and cried. He cannot move his right lower leg. The mother had a hard time explaining how the boy sustained the injury. She said he had several fractures in the past. His father also has history of multiple fractures and hearing loss. On examination, the boy's height is at the 5th percentile, he has normal motor and sensory examination of the right foot and ankle with strong pulses, slight swelling of the right leg with tenderness at the tibia. You suspect a right tibial fracture. Which of the following is the most likely underlying diagnosis? AChild abuse BOsteogenesis imperfecta COsteomalacia DRickets

Osteogenesis imperfecta Osteogenesis Imperfecta (Brittle Bone Disease) Genetic connective tissue disorder, autosomal dominant mutation in collagen Sx: multiple fractures with no history of significant trauma, hearing loss, easy bruising PE: blue sclera, increased laxity of joints and skin, short stature, scoliosis, basilar skull deformities ------------------------------------------------------------------ vs Child abuse (A) can present with multiple fractures in various stages of healing, similar to children with moderate to severe types of OI. In the boy in the vignette, however, there is a family history, which is more suspicious for OI. Osteomalacia (C) can present as bone pain, fractures, and alkaline phosphatase elevation. Hearing loss or blue sclerae are not present. Rickets (D) can lead to slow growth, bone deformities, elevation of alkaline phosphatase, defective bone mineralization, and abnormal tooth formation. However, scleral abnormalities and hearing loss typically do not occur.

Which of the following antibiotics is first line in the treatment of acute Group A streptococcal pharyngitis? AAmoxicillin/clavulanic acid BCiprofloxacin CDoxycycline DPenicillin VK

Penicillin VK Which other two antibiotic classes can be used to treat Group A streptococcal pharyngitis in a penicillin-allergic patient? Answer: First-generation cephalosporin or macrolide. Pharyngitis Most commonly caused by viral > bacteria (GAS, S. pyogenes) Centor criteria: estimates probability pharyngitis is streptococcal based on PE and Sx: anterior cervical lymphadenopathy, tonsillar exudate or swelling, fever, absence of cough Modified Centor criteria: +1 for age < 15 years, -1 for age > 45 years Treatment Viral: supportive Bacterial:Penicillin or amoxicillin Patients with penicillin allergy: cephalexin, cefadroxil (avoid in individuals with immediate-type hypersensitivity to penicillin), clindamycin, azithromycin, clarithromycin (resistance of group A strep to these agents is known well and varies geographically and temporally)

A 34-year-old woman presents to the outpatient family practice office for evaluation. During her interview, she reports that she feels depressed and that this has been her baseline for the last eight or nine years. During this time, she has experienced occasional periods of happiness lasting one to two weeks at the most before returning to a sad or depressed state. She reports a generally low self esteem and indecisiveness which has made career advancement difficult. What is the most likely diagnosis? ABipolar I disorder BBipolar II disorder CMajor depressive disorder DPersistent depressive disorder

Persistent depressive disorder

A 55-year-old woman with a history of obesity and rheumatoid arthritis that is managed with NSAIDs presents after a gradual onset of numbness in her hands over several weeks. She reports the pain is worse at night after she comes home from work as a court reporter. She reports no fever, recent illness, or trauma to the area. Her vital signs are within normal limits. On exam, she has numbness, paresthesia, and pain in the thumb, index, and long fingers. She is worried because her brother had numbness in his right hand last month and was told he had a stroke. Which of the following will help to confirm her diagnosis? AAnteroposterior and lateral films of the bilateral forearms BFinkelstein test CFroment sign DPhalen maneuver

Phalen maneuver dx: carpel tunnel

A 35-year-old woman presents with dark brown patches of skin on her upper lip and forehead. She first noticed the lesions last year while she was pregnant, but they have persisted. What is the most important intervention to reduce worsening of this skin manifestation? ACamouflaging agents BExfoliating agents CPhotoprotection DTopical corticosteroids

Photoprotection dx:Melasma Commonly presents in women who are pregnant or using oral contraceptives Increased deposition of melanin on sun-exposed skin commonly caused by hormonal changes PE: dark, irregular, well-demarcated hyperpigmented macules or patches on face Treatment is sunscreen and sun avoidance During pregnancy, it is called chloasma

A 58-year-old woman presents with progressively worsening muscle weakness. Physical exam reveals weakness of the bilateral hip flexors. Creatine kinase and aldolase are elevated. A muscle biopsy shows primary inflammation with the CD8/MHC-I complex and no vacuoles. Which of the following is the best initial treatment? AMethotrexate BPlasmapheresis CPrednisone DTacrolimus

Prednisone Polymyositis Sx: progressive weakness of 3 months' duration PE: diffuse tenderness of proximal muscles in the shoulder girdle and pelvic girdle without rash or joint swelling and neurologic examination yields normal results Labs: positive anti-Jo, anti-SRP Tx: corticosteroids

A 45-year-old man presents to the office complaining of a "racing heart", palpitations, increased sweating, and headaches for the past three months. He denies any new life stressors and history of anxiety or panic attacks. He is not currently on any medications. The patient reports his father experienced similar symptoms when he was around the same age and was ultimately diagnosed with a tumor on his adrenal gland. Patient's blood pressure in the office is 164/98 mm Hg and his pulse rate is 88 beats per minute with a regular rhythm. Physical examination reveals a diaphoretic, well developed man without focal neurological deficits. TSH and T3/T4 levels were all reported within normal limits. Which of the following diagnostic tests would be the most appropriate next step for this patient? A24-hour urine fractionated metanephrines and catecholamines BCT scan CGenetic testing DPlasma fractionated metanephrines

Plasma fractionated metanephrines initial biochemical test performed for a patient who is considered high risk for a pheochromocytoma (family history, familial tumor syndrome, history of previously resected pheochromocytoma, or presence of adrenal mass found incidentally). These types of tumors are often associated with familial tumor disorders, such as multiple endocrine neoplasia type 2 (MEN2) and von Hippel-Lindau (VHL) syndrome. The triad associated with pheochromocytoma include palpitations or tachycardia, episodic headaches, and diaphoresis. Patients have a history of hypertension. ---------------------------------------------------------- vs 24-hour urine fractionated metanephrines and catecholamines (A) is incorrect as this patient is at high risk of a pheochromocytoma given his family history. A 24-hour urine fractionated metanephrines and catecholamines test would be performed in a patient who is at low risk for a pheochromocytoma. CT scan (B) would not be the initial test in this patient as the initial biochemical test should be performed prior to imaging studies in patients where pheochromocytoma is the suspected diagnosis. Genetic testing (C) is indicated in patients with a positive family history of familial genetic disorders (such as MEN2, von-Hippel-Lindau syndrome, or neurofibromatosis type 1) or whose imaging results are suggestive of a familial disorder.

A 14-year-old boy presents with right eye redness for 1 day. Examination reveals conjunctival injection, yellow discharge, and crusting. His visual acuity is normal, intraocular pressure is 12 mm Hg, and there is no fluorescein uptake. What management is indicated? AAcetazolamide BHomatropine drops CPolymyxin B plus trimethoprim drops DTimolol drops

Polymyxin B plus trimethoprim drops dx:acute conjunctivitis ------------------------------------------------------------- vs Acetazolamide (A) may be useful in treating acute angle closure glaucoma as it can reduce intraocular pressure. Homatropine drops (B) can be used to induce mydriasis. Timolol drops (D) are used in the emergent treatment of acute angle closure glaucoma.

A 15-year-old girl presents with generalized abdominopelvic pain that occurs every month after her regular, nonpainful menses. The pain, which is associated with headaches, bloating, and depressed mood, begins 18 days after the last day of menstruation. She also reports cyclic ankle swelling but does not report a history of increased salt intake. Which of the following is the most likely diagnosis? AAmenorrhea BDysmenorrhea CPremenstrual dysphoric disorder DPremenstrual syndrome

Premenstrual Syndrome 1-2 weeks prior to menstrual cycle Sleep disturbances, decreased focus, emotional lability, breast tenderness, or HA that resolves after menses begins Treatment is decreased caffeine intake, exercise, stress reduction, NSAIDs, SSRIs, OCPs Symptoms do not hinder personal or professional life (unlike premenstrual dysphoric disorder)

A 34-year-old G5P4 woman at 24 weeks gestation presents to the emergency department with vaginal bleeding. A transabdominal ultrasound done in the emergency department shows the placenta overlying the cervical os. Which of the following is a risk factor for this condition? AMaternal hypertension BMaternal trauma CPremature rupture of membranes DPrior dilation and curettage

Prior dilation and curettage dx: Placenta Previa Painless vaginal bleeding Most often during the third trimester Diagnosis is made by ultrasound (transvaginal > transabdominal) Do not do a digital vaginal exam Delivery: cesarean section at 36 0/7-37 6/7 gestation . Risk factors include previous placenta previa, multiple gestation, multiparity, prior cesarean section, and previous intrauterine surgery, including dilation and curettage.

Which of the following is a medical treatment for congenital long QT syndrome? AFlecainide BPropafenone CPropranolol DQuinidine

Propranolol Patient presents with syncope, seizure, palpitations Men: QT interval > 440 msec Women: QT interval > 460 msec More commonly caused by medications > familial prolongation, low Mg, K, Ca Treatment Congenital: beta-blocker, cardiology consult, consider genetic testing and counseling Acquired: stop offending medications, correct electrolyte disturbances, IV magnesium or pacing for torsades de pointes

A 23-year-old man presents to the office with his parents for concerns about mood changes. The parents state that the patient began going through periods of extreme energy about 1 year ago. During these episodes, the patient will be very irritable, sleep < 4 hours per night, be very talkative, have racing thoughts, and have thoughts of buying several mansions. He is unable to function at work during these episodes, which often last for more than 1 week and are occurring every few months. He currently has been in an episode of extreme energy for 2 weeks. His parents also note episodes of sadness and hopelessness that have occurred a few times in the past year as well. His vital signs show a HR of 88 bpm, RR of 17/min, oxygen saturation of 98% SpO2, BP of 117/69 mm Hg, and T of 98°F. Urine drug screen is negative. The patient is disheveled on exam, and he begins talking about unrelated subjects to you with very pressured speech. Based on the patient's suspected diagnosis, which medication is appropriate first-line management? ABupropion BCarbamazepine CParoxetine DQuetiapine EVenlafaxine

Quetiapine dx:Bipolar disorder First-line pharmacotherapy for acute mania in bipolar I disorder includes quetiapine. Other first-line agents include lithium, aripiprazole, and risperidone. Quetiapine is a second-generation or atypical antipsychotic. Quetiapine has been shown to have a 50-60% reduction in mania cycles when treated with doses of 200-800 mg per day. Its mechanism of action is through antagonism at receptors for dopamine and serotonin. --------------------------------------------------------------- vs Paroxetine (C) is an SSRI. As an SSRI, it is considered the first line in the treatment of nonbipolar major depression. It can also be used in the treatment of anxiety.

A 17-year-old male is seen in the clinic to evaluate a painless testicular mass. Ultrasound reveals a solid mass with calcifications and indistinct margins that is worrisome for testicular cancer. Which of the following is used to make a definitive diagnosis? AChest radiograph BRadical inguinal orchiectomy CSerum tumor marker measurement DTesticular biopsy

Radical inguinal orchiectomy dx:Testicular Cancer Risk factors: cryptorchidism, age 15-35 Sx: testicular lump PE: painless, hard, fixed mass Labs: beta-hCG, AFP, or LDH may be elevated based on tumor type Diagnosis starts with ultrasound Tx: radical inguinal orchiectomy usually curative, radiation therapy or platinum-based chemo if metastatic ------------------------------------------------------------------- vs Testicular biopsy (D) is not performed due to concern that it may result in tumor seeding. Serum tumor markers (C) are not sufficiently sensitive or specific to establish the diagnosis of testicular cancer in the absence of histologic confirmation. Although serum tumor markers are helpful at the time of initial diagnosis of a testicular cancer and for prognostication, their main utility is for subsequent follow-up of disease status after primary treatment.

A full-term neonate is brought to the ED for constant crying for the last 3 hours. In the ED, he sleeps quietly in his mother's arms. He cries when you examine him but is immediately consoled when he is swaddled and held. His exam is normal. What is the most appropriate next step in his management? ACT scan of the head to rule out intracranial hemorrhage BLumbar puncture to rule out serious bacterial infection CReassurance DSkeletal survey for abuse

Reassurance An easily consoled infant without a source of crying after a thorough history and physical exam can be discharged with parental reassurance.

A 30-year-old woman with no past medical history presents to the emergency department with substernal chest pain for two hours. It is not worse with exertion and was not relieved by sublingual nitroglycerin. She reports some mild nausea. She does not smoke cigarettes or use any illicit drugs. Her family history includes a grandmother who died of a myocardial infarction at 84 years old. Labs in the emergency department are unremarkable. Point of care troponin is negative, and ECG reveals sinus rhythm. What is your next step in management? ADobutamine stress test BExercise stress test CReassurance DStress echocardiography

Reassurance is most appropriate for this patient as her pretest probability for coronary heart disease it is very low.

A 12-year-old girl is brought to the clinic for evaluation of hip pain. She describes the pain as dull, non-radiating and aching pain on the right hip that is increased with physical activity and slightly relieved with rest. The pain has been intermittent for the past four weeks. She denies any trauma. On physical exam, weight is at the 95th percentile, the right anterior hip is tender to palpation, and there is decreased internal rotation, abduction, and flexion of the right hip. The rest of the examination is normal. Radiographs show a blurring of the junction between the metaphysis and the growth plate on AP view and step-off of the epiphysis on the femoral neck on frog-leg view. Which of the following is the next best step in management? AAdvise rest and close follow-up BObtain magnetic resonance imaging CPrescribe an anti-inflammatory medication DRefer to an orthopedic surgeon

Refer to an orthopedic surgeon Slipped Capital Femoral Epiphysis (SCFE) Patient will be a boy 12-16 years old with obesity Progressive limp and hip or knee pain PE will show loss of hip internal rotation X-ray will show scoop of ice cream slipping off cone Diagnosis is made by AP and frog-leg lateral X-rays Treatment is non-weight-bearing and urgent orthopedic consultation

What other evaluation should a febrile infant with first urinary tract infection be referred to undergo? AIntravenous pyelogram BNuclear scanning with technetium-labeled dimercaptosuccinic acid CRenal and bladder ultrasonography DVoiding cystourethrography

Renal and bladder ultrasonography The 2011 American Academy of Pediatrics UTI Clinical Practice Guideline, Action Statement 5, states that febrile infants with first UTI should undergo renal and bladder ultrasonography (RBUS) to detect anatomic abnormalities that may require further evaluation and intervention. Host Factors That Influence the Predisposition to UTI in Children------image---->>>>> The RBUS should be done during the first two days of treatment to identify serious complications such as renal or perirenal abscesses or pyelonephrosis if the patient's clinical illness is unusually severe or is not clinically improving. However, if the patient demonstrates significant improvement, then the RBUS should be performed after resolution of the acute illness. -------------------------------------------------------------

An 18-month-old previously healthy girl presents to the clinic, brought in by her parents, due to a persistent fever over the past 2 days with a maximum temperature of 102.7°F. Her parents report that she is not toilet trained yet, and she has had two prior similar episodes since age 13 months. Physical examination reveals suprapubic and costovertebral tenderness. Dipstick analysis shows positive leukocyte esterase. Urine culture is pending, but a prior urine culture revealed significant growth of Klebsiella. Kidney ultrasound reveals dilation of the drainage system of the renal calyces, infundibula, and pelvis. Which of the following is the most likely complication secondary to the suspected condition? AAnasarca with ascites BBladder outlet obstruction with bladder wall muscular hypertrophy CInflammation of the renal glomerulus DObstruction from uric acid stones ERenal scarring with subsequent chronic kidney disease

Renal scarring with subsequent chronic kidney disease dx:Vesicoureteral Reflux Hydronephrosis on U/S Primary: incompetent UVJ closure Non-toilet-trained children with febrile UTI, prenatal Dx Secondary: ↑ voiding pressure causing closure dysfunction Toilet-trained children with UTI VCUG to Dx and grade severity Surveillance → prophylactic Abx → surgery Complications of VUR commonly include recurrent UTIs or pyelonephritis and loss of renal parenchyma due to renal scarring with subsequent chronic kidney disease. What is the definition of nephrotic-range proteinuria? Answer: Urinary protein excretion > 3.5 g in 24 hours. --------------------------------------------------------------- vs Inflammation of the renal glomerulus (C) is termed glomerulonephritis and presents with edema, hypertension, reddish-brown to brown-colored urine, a rising serum creatinine, and an active urinary sediment (i.e., red cell casts and dysmorphic red blood cells). Acute and chronic cases are primarily attributed to immunologic etiologies and are not a complication of VUR. Bladder outlet obstruction with bladder wall muscular hypertrophy (B) is a complication of posterior urethral valves, which are obstructing membranous folds within the lumen of the posterior urethra that aberrantly develop during the male embryonic stages. While the condition is commonly diagnosed using prenatal ultrasound, the postnatal presentation includes recurrent UTIs, abdominal distention, failure to thrive, and incontinence in older boys.

A 25-year old woman presents to her primary care physician for a routine annual visit. Her last Papanicolaou test was three years ago. She smokes cigarettes and is sexually active. Her mother was diagnosed with ovarian cancer at age 50. According to the current guidelines, which of the following should be offered to the patient? AMammography BMeningococcal vaccine COvarian ultrasound DPapanicolaou test

Routine screening for cervical cancer with the Papanicolaou test (Pap smear) is recommended for all women aged 21 years or older, at least every 3 years.

A 31-year-old woman presents with thinning of her hair over the past month and fatigue. She says she thinks she is pulling out more hair than normal when she grooms. The patient is 4 months postpartum, and her labor and delivery and pregnancy were uncomplicated. Physical examination reveals conjunctival pallor and dry skin. A hair-pull test extracts 25-30 hairs with white bulbs at the ends. Which of the following is the most appropriate test to reveal the underlying cause? AAnti-double-stranded DNA BScalp biopsy CSerum ferritin DSerum testosterone ETransvaginal ultrasound

Serum ferritin dx;Telogen Effluvium The patient in this scenario has signs and symptoms of iron deficiency anemia with a clinical history of recent childbirth, making telogen effluvium due to iron deficiency most likely. Common causes: child birth, major illness or surgery, stress, medications, malnutrition ------------------------------------------------------------- vs anti-double-stranded DNA test (A) is not the best test to order in this patient as other symptoms would likely exist in a patient with alopecia due to systemic lupus erythematosus (SLE). Additionally, SLE is often more likely to cause cicatricial alopecia, or alopecia from scarring, which this patient does not have. A scalp biopsy (B) is occasionally used in diagnosing other conditions, such as alopecia areata, but this patient does not have associated symptoms. A serum testosterone (D) would be more appropriate if this patient showed a hair loss pattern characteristic of androgenic alopecia, which can be associated with polycystic ovarian syndrome (PCOS). Androgenic alopecia tends to cause gradual hair loss at the calvarium, lateral sides of the widow's peak and on the crown. A transvaginal ultrasound (E) is useful for individuals with suspected PCOS.

A 70-year-old man with a history of chronic kidney disease, hypertension, and type 2 diabetes mellitus presents to his nephrologist's office for follow-up. His vital signs are HR 84 bpm, RR 18/min, BP 124/82 mm Hg, T 98.3°F, SpO2 98%, and BMI 30 kg/m2. The patient is currently on hemodialysis 3 days per week. His medications include insulin aspart, insulin glargine, linagliptin, metoprolol, and lisinopril. Laboratory results show an estimated glomerular filtration rate of 20 mg/mmol, blood urea nitrogen of 25 mg/dL, creatinine of 1.9 mg/dL, serum calcium of 8.6 mg/dL, serum phosphate of 6.0 mg/dL, and magnesium of 2.4 mg/dL. His serum phosphate levels remain elevated despite implementing phosphate restrictions in his diet. He is asymptomatic and reports no symptoms of muscle cramps or weakness, paresthesias, or circumoral numbness. Which of the following is the best next step in managing his condition? ACalcium citrate BContinue dietary reduction of phosphate CHemodialysis DIntravenous calcium ESevelamer

Sevelamer is the appropriate treatment of choice in asymptomatic patients with chronic hyperphosphatemia. The most common causes of hyperphosphatemia are transcellular shift of phosphate and decreased renal clearance due to chronic kidney disease (CKD). phosphate concentration of 3.5-5.5 mg/dL. Patients are generally asymptomatic but may present with symptoms of hypocalcemia such as muscle cramps or spasms, paresthesias, circumoral numbness, or QTC prolongatio Phosphate binders are divided into two groups: calcium-containing binders, such as calcium carbonate and calcium acetate, and non-calcium-containing binders, such as sevelamer and lanthanum. Non-calcium-containing binders are preferred over calcium-containing binders, which may be associated with increased serum calcium levels, vascular calcification, and bone disease. Studies have shown that increasing the dose of dialysis above the recommended dose does not improve clinical outcomes. ----------------------------------------------------------------- vs Calcium citrate (A) may be used as a phosphate binder. It should be avoided in this patient due to his history of CKD and increased risk of aluminum neurotoxicity, as citrate is associated with intestinal aluminum absorption. Citrate maintains aluminum soluble in the intestines and combines with calcium in the intestinal lumen. As free calcium decreases, the permeability of cellular tight junctions increases and increases aluminum absorption. Continuing dietary reduction of phosphate (B) may help reduce serum phosphate levels in patients with chronic hyperphosphatemia. In patients with persistently elevated serum phosphate levels, the use of phosphate binders is recommended, regardless of dietary restrictions. It is not recommended to increase the hemodialysis (C) dose in patients already undergoing hemodialysis, as it has not been shown to improve clinical outcomes. Intravenous calcium (D) is recommended in patients who present with symptomatic hypocalcemia, not hyperphosphatemia. Hypocalcemia may present with paresthesias of the hand and feet, perioral numbness, and muscle cramping, or in severe cases with carpopedal spasms, laryngospasms, and seizures.

A 62-year-old man presents to the emergency department with right sided facial pain. His pulse is 99 beats/min, blood pressure is 108/62 mm Hg, and temperature is 102.1°F. On exam, there is erythematous, pre-auricular swelling that extends to the angle of the mandible. The area is exquisitely tender and purulent material can be expressed from Stensen's duct. Which of the following is the most likely diagnosis? ALudwig's angina BOral candidiasis CPeritonsillar abscess DSuppurative parotitis

Suppurative parotitis Patient will be older History of dehydration or intubation Pain and tenderness with trismus and dysphagia PE will show a firm, erythematous swelling of the pre- and postauricular areas that extends to the angle of the mandible Most commonly caused by Staphylococcus aureus Treatment is intravenous antibiotics Complications include respiratory obstruction, osteomyelitis, and septicemia. -------------------------------------------------------------- vs Ludwig's angina (A) is an infection of the sublingual and submaxillary spaces. It is caused by infection of the mandibular teeth. Ludwig's angina presents with edema and erythema of the floor of the mouth, chin, and upper neck. Treatment involves airway management and intravenous antibiotics. Peritonsillar abscess (C) is usually a complication of tonsillitis or pharyngitis. Patients with peritonsillar abscess present with a severe sore throat, odynophagia, trismus, and a muffled voice. Physical exam reveals a deviated uvula and soft palate.

A 31-year-old woman with a history of hypertension and asthma presents to her OB/GYN's office. The patient is planning a pregnancy within the next few months. She takes lisinopril daily and albuterol as needed and is not on any other medications. Her vital signs are a HR of 65 bpm, RR of 17/min, SpO2 of 99% on room air, BP of 118/80 mm Hg, T of 97.3°F, and BMI of 28.6 kg/m2. Her most recent laboratory findings consist of the following values: A1C of 5.9%, potassium of 3.8 mmol/L, and creatinine at 0.8 mg/dL. Which of the following is the most appropriate management for this patient's hypertension? AContinue current treatment BSwitch to amlodipine CSwitch to chlorthalidone DSwitch to labetalol ESwitch to nifedipine ER

Switch to nifedipine ER The following medications are acceptable for the treatment of hypertension in pregnancy: methyldopa, nifedipine, and labetalol. #LMNOP ------------------------------------------------------------- vs Continuing the current treatment (A) is not appropriate as ACE inhibitors, such as lisinopril, are not safe in pregnancy. These medications are teratogenic in the first trimester, and thus it is important to discontinue use if the patient has plans of becoming pregnant within the next few months. Amlodipine (B) and chlorthalidone (C) are not considered safe during pregnancy. Switching to labetalol (D) is not the most appropriate treatment for this patient, given that her heart rate is 65 bpm during the visit. Labetalol is a beta-blocker that would further reduce the patient's heart rate. In addition, beta-blockers should be avoided in patients with asthma.

A 52-year-old man with obesity presents to your office with right foot pain. He reports that his alcohol consumption has been increasing over the past 5 years, and he frequently eats surf and turf at his favorite restaurant. He takes daily aspirin and simvastatin. Physical exam reveals redness, warmth, inflammation, and exquisite tenderness to palpation of his right great toe. Which of the following provides the most definitive diagnosis? ABlood test BClinical diagnosis CSynovial fluid analysis DUltrasound

Synovial fluid analysis dx; Gout "podagra" Acute: NSAIDs, steroids, colchicine Chronic: allopurinol (first line), febuxostat, probenecid Can be triggered by loop and thiazide diuretics

A 29-year-old woman is concerned with a rash. On inspection, you appreciate a central facial rash. You notice that the nasolabial folds are not affected. You also detect alopecia and lip crusting. The neck, chest and trunk are void of cutaneous abnormalities. The dorsal digits, and other extensor surfaces, are not affected. Which of the following is the most likely diagnosis? ADermatomyositis BPsoriasis CRosacea DSystemic lupus erythematosus

Systemic lupus erythematosus ---------------------------------------------------------------- Alopecia and hemorrhagic crusting of the lips occurs more commonly in SLE than rosacea (C).

A 53-year-old man is brought into the ED confused and disoriented. His partner reports he was feeling weak and nauseous and had multiple episodes of vomiting. On further questioning, he also reports an increase in hiccuping, a metallic taste in his mouth, and general pruritus. His blood pressure is 160/90, and he is tachypneic at 30 breaths per minute. Heart rate is 94 beats per minute. He is afebrile. On exam, dependent edema is noted. Multiple screening tests are sent, and they are notable for an elevated serum creatinine level of 2.1 mg/dL with an estimated glomerular filtration rate of 50 mL/min and blood glucose of 280 mg/dL on metabolic panel. His urinalysis is positive for protein. What is the most likely underlying cause for his presenting symptoms? ADiabetes mellitus BNephrolithiasis CNephrotic syndrome DPolycystic kidney disease

The most common causes of chronic kidney disease are diabetes mellitus (type 2) and hypertension. In addition to being the leading cause of chronic kidney disease, diabetes is also the most common cause of end-stage kidney disease. ---------------------------------------------------------------- vs Nephrolithiasis (B), or kidney stones, is associated with nausea and vomiting, but the hallmark is an acute onset of severe, colicky flank pain. A urinalysis would most likely reveal hematuria and an altered pH, with no suggestion of kidney insufficiency. Nephrotic syndrome (C) can also be due to underlying diabetes mellitus, which leads to nodular glomerulosclerosis. Patients will present with generalized edema and foamy urine. Laboratory workup will reveal proteinuria, hypoalbuminemia, and hyperlipidemia. Polycystic kidney disease (D) is an inherited progressive cystic dilation of the renal tubules that can be accompanied by cysts in the spleen, liver, and pancreas. The most common presenting symptoms are pain and hematuria. Hypertension may also be noted and can contribute to kidney damage, resulting in end-stage kidney disease.

A 25-year-old G2P1 woman presents to your office at 32 weeks gestation with a complaint of severe itching, particularly on the palms of her hands and the soles of her feet. Lab results reveal elevated bile acids. Regarding this disease, which of the following statements is most correct? AAminotransferases are low BDisease recurrence is rare in subsequent pregnancies CThe treatment of choice is cholestyramine DThere is an increased risk for fetal demise

There is an increased risk for fetal demise dx:Intrahepatic Cholestasis of Pregnancy Treatment • Ursodeoxycholic acid In intrahepatic cholestasis of pregnancy do the number of adverse fetal outcomes increase as serum bile acid levels rise? Answer: Yes.

A patient presents to the emergency department after a motor vehicle collision with a complaint of dyspnea and pleuritic chest pain. Physical examination reveals diminished breath sounds on the left and a chest X-ray demonstrates mediastinal shift to the right. Which of the following is the most appropriate first-line treatment for this condition? AObservation BPleurodesis CThoracentesis DThoracostomy

Thoracostomy Tension Pneumothorax History of chest trauma PE will show diminished or absent breath sounds, tracheal deviation away from the side of the injury, hypotension, jugular venous distension Diagnosis is made clinically Treatment is needle decompression of the chest in the 2nd intercostal space in the midclavicular line or 4th or 5th intercostal space in the anterior axillary line, followed by chest tube insertion Finger thoracostomy followed by chest tube insertion may be faster

You are treating a 15-year-old patient who is admitted to the hospital with a severe flare of ulcerative colitis. You have prescribed antibiotics, corticosteroids, and a biologic medication only. The acute phase of this illness places this patient most at risk for which of the following? AColon cancer BPancreatitis CPeptic ulcer DToxic megacolon

Toxic megacolon Ulcerative Colitis Sx: bloody diarrhea, crampy abdominal pain, tenesmus PE: continuous mucosal inflammation, always involving the rectum, absence of perianal involvement (prevalent in Crohn) Extraintestinal findings: uveitis, erythema nodosum, peripheral arthritis, sacroiliitis, ankylosing spondylitis Tx:options depend on severity and location of disease Mild-moderate: mesalamine, topical or oral steroids, 5-ASA Severe: IV steroids +/- topical steroids initially, then anti-TNF or anti-integrin, colectomy for refractory cases (curative) Complications: toxic megacolon, ↑ colon cancer risk

A 58-year old postmenopausal woman presents with painless vaginal bleeding. Her last menses occurred 5 years ago. She reports that her Papanicolaou smears have always been normal; the last one was obtained a year ago. Which of the following is the next step in management? AColposcopy with endocervical curettage BHysteroscopy CRepeat Papanicolaou smear DTransvaginal ultrasound

Transvaginal ultrasound is the preferred initial diagnostic test of choice to evaluate painless vaginal bleeding in a postmenopausal patient in order to rule out endometrial (uterine) carcinoma Endometrial Cancer Peak incidence is in postmenopausal patients between age 60-70 years Most common type is adenocarcinoma Endometrial intraepithelial neoplasia is a precursor lesion Risk factors: nulliparity, obesity, unopposed estrogen, tamoxifen (postmenopausal patients) Sx: abnormal vaginal bleeding Dx: endometrial biopsy Tx: total hysterectomy and bilateral salpingo-oophorectomy Pelvic and para-aortic lymphadenectomy for comprehensive staging ------------------------------------------------------------------- vs Colposcopy (A) is not indicated given the patient's normal Papanicolaou smear. Hysteroscopy (B) should be performed in the operating room if an office endometrial biopsy is not obtainable. A repeat Papanicolaou smear (C) is used to diagnose cervical cancer but not endometrial cancer. This patient also has a history of negative smears in the past, including one only a year ago.

Which of the following antibiotic choices provides broad-spectrum coverage for gram-positive organisms only? AAztreonam BErtapenem CMeropenem DVancomycin

Vancomycin Inhibits bacterial cell wall synthesis by blocking glycopeptide polymerization through binding tightly to D-alanyl-D-alanine portion of cell wall precursor A serum trough level of 15-20 mg/L is considered therapeutic for most infections in critically ill patients. Vancomycin is typically administered parenterally, but oral formulations are required for the treatment of Clostridioides difficile infections. -------------------------------------------------------------- vs Aztreonam (A) is a monobactam and provides gram-negative coverage. Ertapenem (B) and meropenem (C) are carbapenems. They provide broad-spectrum coverage of gram-negative, gram-positive, and anaerobes, rather than gram-positive coverage only.

A 22-year-old woman presents to urgent care due to genital lesions for the past 2 days. She states the lesions are painful and tender. She reports no history of similar problems and no abdominal pain or changes in her menstrual cycle. She has a history of gonorrhea 2 years ago but, otherwise, has no significant medical history. She reports intercourse with multiple men and uses oral contraceptives for birth control. Her only medication is her oral birth control pill. Her vitals include a BP of 120/76 mm Hg, HR of 78 bpm, RR of 13/min, T of 99.9°F, and SpO2 of 100% on room air. During the genital exam, you note inguinal lymphadenopathy and multiple groups of small umbilicated vesicles with underlying erythema and ulcerations. What is the most sensitive diagnostic test for the most likely diagnosis?😨😨😨😨 ADirect fluorescent antibody BSerologic testing CTzanck smear DViral culture EViral polymerase chain reaction assay

Viral polymerase chain reaction assay Genital Herpes Simplex Sx: painful genital rash, may be asymptomatic PE: grouped erythematous shallow cluster of vesicles and lymphadenopathy Labs: multinucleated giant cells on Tzanck smear (poor sensitivity) Dx: tissue PCR or viral culture Most commonly caused by herpes simplex virus (HSV) type 2, but HSV-1 infections are increasing in frequency Tx: acyclovir Pregnancy: acyclovir or valacyclovir for 7-10 days after primary infection and from 36 weeks to delivery -------------------------------------------------------------- vs A Tzanck smear (C), done by scraping active lesions, can help confirm the diagnosis of genital herpes, but it has a low sensitivity and specificity. PCR assay is the most sensitive test for HSV infection.

A patient presents complaining of hearing loss on the left. Examination of the right ear is normal but the examination of the left ear shows cerumen impaction. Which of the following combination of examination findings is consistent with this presentation? AWeber test localizes to the left ear; Rinne test shows air conduction greater than bone conduction on both sides BWeber test localizes to the left ear; Rinne test shows bone conduction greater than air conduction on the left side and air conduction greater than bone conduction on the right side CWeber test localizes to the right ear; Rinne test shows air conduction greater than bone conduction on both sides DWeber test localizes to the right ear; Rinne test shows bone conduction greater than air conduction on the right side and air conduction greater than bone conduction on the left side

Weber test localizes to the left ear; Rinne test shows bone conduction greater than air conduction on the left side and air conduction greater than bone conduction on the right side dx: conductive loss Hearing Loss Conductive Weber: localizes to affected ear Rinne: abnormal (BC > AC) Most commonly caused by otitis media, serous otitis, and cerumen impaction Sensorineural Weber: localizes to unaffected ear Rinne: normal (AC > BC) Most commonly caused by excessive noise exposure, drugs, and normal aging

A 58-year-old man with chronic hypertension presents to the ED with 10/10 tearing chest pain that radiates to the back. All you can gather from him is that he also has some type of "collagen disorder" and diabetes. A chest radiograph reveals a widened mediastinum. As you prepare for a transesophageal echocardiogram, you would most likely start which of the following medications as a first-line agent? AClonidine BLabetalol CLisinopril DNitroprusside

labetalol First-line therapy is an intravenous beta-blocker, such as labetalol or esmolol (preferred due to its easy titratability), followed by vasodilators such as nicardipine or nitroprusside. Surgery is indicated for Type A dissections that involve the aortic arch. dx: aortic dissection Risk factors: advancing age, male sex, HTN, Marfan syndrome Sx: acute onset of "ripping" or "tearing" chest pain or back pain PE: asymmetric pulses or SBP difference of > 20 mmHg CXR: widened mediastinum Dx: CT angiography or transesophageal echocardiogram (TEE) Treatment: reduce BP and HR (beta-blockers), pain control, emergency surgery (Type A dissection) IV Labetalol or esmolol Type A: involves ascending aorta Type B: involves only descending aorta . A new aortic insufficiency murmur is more common in a proximal dissection which involves the aortic arch. Evaluation includes chest radiograph, chest CTA, and transesophageal echocardiogram.

An elderly patient is diagnosed with wet age-related macular degeneration. Which of the following medications is used to treat this disease? AAdalimumab BEtanercept CInfliximab DRanibizumab

ranibizumab The initial agent considered was bevacizumab, which is now more commonly used in cancer treatment. A smaller fragment of bevacizumab, ranibizumab, is now currently used for intraocular injection therapy. Wet age-related macular degeneration causes a rapidly progressive central visual impairment. It is mainly due to neovascularization and blood accumulation between the choroid and retina. This proliferation of blood vessels is driven by vascular endothelial growth factor (VEGF) Macular Degeneration Patient will be older Bilateral, gradual central field vision loss PE Dry (85% of cases): atrophic changes and yellow retinal deposits (drusen spots) Wet: vascular changes Diagnosis is made by characteristic findings on dilated eye examination Most common cause of blindness in the older population --------------------------------------------------------------- vs Adalimumab (A), etanercept (B), and infliximab (C) are all biologic medications used in treating rheumatoid arthritis, not macular degeneration.


Kaugnay na mga set ng pag-aaral

Chapter 10 . Emerging into Adulthood, and Constructing an Adult Life

View Set

Chapter 6, 7, 8, 9 personal finance

View Set

med surg nursing for exam 1 practice questions

View Set

Integumentary System test Review A&P

View Set